Difference between revisions of "2014 AMC 12A Problems/Problem 4"
(Created page with "==Problem== Suppose that <math>a</math> cows give <math>b</math> gallons of milk in <math>c</math> days. At this rate, how many gallons of milk will <math>d</math> cows give in ...") |
(Redirect to 2014 AMC 10A Problems/Problem 6) |
||
(3 intermediate revisions by 2 users not shown) | |||
Line 1: | Line 1: | ||
− | + | #REDIRECT[[2014 AMC 10A Problems/Problem 6]] | |
− | |||
− | |||
− | |||
− | |||
− | |||
− | |||
− | |||
− |
Latest revision as of 13:19, 8 February 2014
Redirect to: